Ajastako muka ei saa otetta?

Aloittaja Joksa, 18.09.2013, 18:22:39

« edellinen - seuraava »

mistral

#90
Lainaus käyttäjältä: ketarax - 25.03.2017, 11:50:34
Kuvailuilassasi esiintyy vieläkin gravitaatiokuoppia (tää on GR:ää, gravitaatiokuopat newtonia), nousukykyä (vieläkin rakettifotoni?) ja huolimatonta kielenkäyttöä kuten 'fotonin pysäyttäminen' (ei ne pysähdy) jotka tekee vähän vaikeaksi arvioida "mitä mielessäsi todella liikkuu", mutta kyllä minä silti luulisin ymmärtäneeni mitä tarkoitit, enkä ole siitä varsinaisesti eri mieltä.

'Fotonin pysäyttäminen' tarkoittaa Maan vinkkelistä katsottuna. On 2 eri mekanismia (jotka voi olla myös yhtä aikaa voimassa) jotka voi 'pysäyttää' fotonin, ajan pysähtyminen ja ääretön gravitaatio. Tai ääretön gravitaatio on ensinnäkin singulariteetin ulkopuolella ristiriitainen juttu ja toiseksi, se ei pysäyttäisi fotonia kirjaimellisesti vaan kääntäisi sen suunnan 180 astetta ja kolmanneksi ääretön nostaisi sen massan äärettömäksi mikä on mahdotonta (?).

Itse kallistun vaan sen kannalle että pysähtynyt aika 'pysäyttää' fotonin.

Mutta kun puhutaan pysähtyneestä ajasta, en vaan ymmärrä miksi se pysähtyy massan pakonopeuden tasossa. Tämä taas johtaa kysymykseen, miksi massan ja valon pakonopeus on samat. Tämä taas johtaa kysymykseen, miksi punasiirtymä kasvaa äkkijyrkästi äärettömäksi lähestyttäessä horisonttia mutta massalle käytetään ei-äkkijyrkkää kenttää. Siis jos tykki ampuu c nopeudella kuulan horisontista, on kuulan massaenergia kylläkin ääretön mutta kenttä josta se nousee, ei ole äkkijyrkkä vaan voimistuu johdonmukaisesti kohti singulariteettia, horisontti on vielä (tässä tapauksessa) 3km päässä singulariteetista joten ääretön gravitaatio on vasta siellä. Sensijaan fotonille kenttä on äkkijyrkkä, miksi näin?

Varmaan joku Valtaoja tai Kurki-Suonio tietää mistä äkkijyrkkyys jo horisontissa johtuu, itse en keksi muuta selitystä kuin aikadilataation. Mutta siinäkin on ristiriitoja.

mistral

Lainaus käyttäjältä: Joksa - 25.03.2017, 12:33:57
Horisonttitasosta fotonin kuuluu lähteä koska sen pakonopeus riittää. Se etäisyys jossa fotoni on vielä havaittavissa voisi katsoa määrittävän sen pääseekö se ma:n gravitaatiokaivon ulkopuolelle vai jääkö se g-kaivoon.

Tässä vaan tekninen kehitys ratkaisisi pääseekö fotoni pakoon. Jos nykyään voidaan mitata vaikkapa 1 miljoonan km aalto ja tulevaisuudessa 10 miljoonan km aalto, se ratkaisisi fotonin kohtalon.

jussi_k_kojootti

Lainaus käyttäjältä: mistral - 25.03.2017, 13:29:17
Tässä vaan tekninen kehitys ratkaisisi pääseekö fotoni pakoon. Jos nykyään voidaan mitata vaikkapa 1 miljoonan km aalto ja tulevaisuudessa 10 miljoonan km aalto, se ratkaisisi fotonin kohtalon.

Miten se sen ratkaisisi?  Miksi ongelmasi ei siirtyisi 100 miljoonan kilometrin aallolle?
jussi kantola / oulun arktos
CG-5 GOTO + KWIQ-guiding + SW80ED  // 10" dobson // canon eos 450d mod & 400d / ASI 120MM
http://astrobin.com/users/jussi_k_kojootti/
http://oulunarktos.fi/

jussi_k_kojootti

#93
Lainaus käyttäjältä: mistral - 25.03.2017, 13:18:33
Varmaan joku Valtaoja tai Kurki-Suonio tietää mistä äkkijyrkkyys jo horisontissa johtuu, itse en keksi muuta selitystä kuin aikadilataation. Mutta siinäkin on ristiriitoja.

"Äkkijyrkkyys johtuu" siitä, että avaruutemme on tietyllä tavalla jäykkä, tietyllä tavalla taipuisa.  Massaenergia taivuttaa, kaareuttaa sitä tietyllä tavalla. Se "äkkijyrkkä" kohta tosi tiheän massaenergian kaareuttamassa avaruudessa on niin monella tapaa mielenkiintoinen, että sille on annettu oma nimi, tapahtumahorisontti.

Sitten sama uudestaan, nyt runsaammalla määrittelyllä/selittelyllä.

"Äkkijyrkkyys johtuu" siitä, että avaruutemme on tietyllä tavalla jäykkä, eli tietyllä tavalla taipuisa.  Massaenergia taivuttaa, kaareuttaa sitä tietyllä tavalla, "tietyn verran tietyillä ehdoilla", ehdoilla kuten etäisyys massaenergiaan, tai massaenergian tiheys (eli massaenergian määrä tilavuusyksikköä kohden).  Se "äkkijyrkkä" kohta tosi tiheän massaenergian kaareuttamassa avaruudessa on niin monella tapaa mielenkiintoinen -- klassinen pakonopeus = c, z = inf, gamma = inf jne. --, että sille on annettu oma nimi, tapahtumahorisontti.

Onko tässä jotain mitä et ymmärrä?  Voisikohan foorumi tässä kohtaa osallistua sen verran keskusteluun, että sanotte reilusti onko selitykseni mielestänne ymmärrettävä, sekava, vai turha?   Onko se mielestänne ja käsittämänne suhteellisuusteorian kuvausta huonosti, kelvollisesti vai hyvin vastaava?  Kaikilla on oma ymmärryksensä "peruskäsitteistä" -- massaenergia, avaruus, ... -- mutta ne huomioiden -- meneekö jakeluun, ostaisitteko ilmaiseksi?  Ja pitäisikö olla aina aivan loppuun asti pedanttinen kohdissa kuten avaruus<->aika-avaruus, vai voiko mielestänne välillä puhua vähän väljemmin?

Arktoksen puheenjohtajana joudun, eli omasta vinkkelistä pääsen :-), välillä puhumaan näistä-kin asioista ns. yleisölle.  Silloinkaan ei voi turvautua matematiikkaan, joten jossain mielessä näitä kirjoituksia voi pitää harjoitteluna.  En varsin epäile tietojani esim. tämän ketjun viimeaikaisten kirjoituksien aiheissa, mutta omasta esityksestäni en sitten voikaan olla niin varma.  Onko liikaa oletuksia lähtötiedoista <=> liian vähän määrittelyjä niistä "peruskäsitteistä"?  Onko kielenkäyttöni liian monimutkaista?  Liian simppeliä?  Lainausmerkkejä ainakin on varmasti liikaa :-S

Yleisö on yleensä tyytyväisenoloista, mutta ehkä ne ei vaan henno sanoa.  Kaipaisin oikeasti jonkinlaista väliarviointia siitä kannattaako tätä ylipäätään jatkaa.  Yleisesti ottaen kyllä kuvittelen, että hiljaisuus on ainakin jonkinasteinen hyväksyntä, ja että virheisiin puututtaisiin.  Niinhän itsekin teen :)
jussi kantola / oulun arktos
CG-5 GOTO + KWIQ-guiding + SW80ED  // 10" dobson // canon eos 450d mod & 400d / ASI 120MM
http://astrobin.com/users/jussi_k_kojootti/
http://oulunarktos.fi/

mistral

Lainaus käyttäjältä: ketarax - 25.03.2017, 20:26:43
Miten se sen ratkaisisi?  Miksi ongelmasi ei siirtyisi 100 miljoonan kilometrin aallolle?

Itse en näe eroa pitkän ja lyhyen aallon geodeesissa, olkoon vaikka linnuradan pituinen. Eli pieni energia ei estä menemästä samaan suuntaan kuin isokin.

mistral

Lainaus käyttäjältä: ketarax - 25.03.2017, 21:07:59
"Äkkijyrkkyys johtuu" siitä, että avaruutemme on tietyllä tavalla jäykkä, eli tietyllä tavalla taipuisa.  Massaenergia taivuttaa, kaareuttaa sitä tietyllä tavalla, "tietyn verran tietyillä ehdoilla", ehdoilla kuten etäisyys massaenergiaan, tai massaenergian tiheys (eli massaenergian määrä tilavuusyksikköä kohden).  Se "äkkijyrkkä" kohta tosi tiheän massaenergian kaareuttamassa avaruudessa on niin monella tapaa mielenkiintoinen -- klassinen pakonopeus = c, z = inf, gamma = inf jne. --, että sille on annettu oma nimi, tapahtumahorisontti.

Ymmärrän tämän laskurin niin että punasiirtymä lähestyy ääretöntä äkkijyrkästi horisontissa.

    d [m]                 f1 [Hz]                   z
1.000000e+12    1.000000e+20   -8.396640e-9 (-0.00)
1.000000e+11    1.000000e+20   4.897159e-9 (0.00)
1.000000e+10    9.999999e+19   1.378351e-7 (0.00)
1.000000e+9    9.999985e+19   1.467214e-6 (0.00)
1.000000e+8    9.999852e+19   1.476091e-5 (0.00)
1.000000e+7    9.998523e+19   1.476881e-4 (0.00)
1.000000e+6    9.985262e+19   1.475990e-3 (0.00)
1.000000e+5    9.855485e+19   1.466337e-2 (0.01)
1.000000e+4    8.786079e+19   1.381642e-1 (0.14)
1.000000e+3    5.028887e+19   9.885114e-1 (0.99)
1.000000e+2    1.809476e+19   4.526461e+0 (4.53)
1.000000e+1    5.808285e+18   1.621679e+1 (16.22)
1.000000e+0    1.839536e+18   5.336155e+1 (53.36)
1.000000e-1    5.818008e+17   1.708801e+2 (170.88)
1.000000e-2    1.839844e+17   5.425244e+2 (542.52)
1.000000e-3    5.818106e+16   1.717772e+3 (1717.77)
1.000000e-4    1.839847e+16   5.434235e+3 (5434.23)
1.000000e-5    5.818107e+15   1.718672e+4 (17186.72)
1.000000e-6    1.839847e+15   5.435135e+4 (54351.35)
1.000000e-7    5.818107e+14   1.718762e+5 (171876.21)
1.000000e-8    1.839847e+14   5.435225e+5 (543522.47)
1.000000e-9    5.818107e+13   1.718771e+6 (1718771.13)
1.000000e-10    1.839847e+13   5.435234e+6 (5435233.71)
1.000000e-11    5.818107e+12   1.718772e+7 (17187720.32)
1.000000e-12    1.839847e+12   5.435235e+7 (54352346.15)
1.000000e-13    5.818107e+11   1.718772e+8 (171877212.16)
1.000000e-14    1.839847e+11   5.435235e+8 (543523470.47)
1.000000e-15    5.818107e+10   1.718772e+9 (1718772130.59)
1.000000e-16    1.839847e+10   5.435235e+9 (5435234713.66)
1.000000e-17    5.818107e+9   1.718772e+10 (17187721314.94)
1.000000e-18    1.839847e+9   5.435235e+10 (54352347145.61)
1.000000e-19    5.818107e+8   1.718772e+11 (171877213158.43)
1.000000e-20    1.839847e+8   5.435235e+11 (543523471465.07)
1.000000e-21    5.818107e+7   1.718772e+12 (1718772131593.33)
1.000000e-22    1.839847e+7   5.435235e+12 (5435234714659.76)
1.000000e-23    5.818107e+6   1.718772e+13 (17187721315942.28)
1.000000e-24    1.839847e+6   5.435235e+13 (54352347146606.30)

Alimmalla rivillä säteilijä on jossain millin miljardisosien(?) päässä horisontista ja Maassa mitataan aallonpituudeksi 54mrd(?) kilometriä. En muista noita suuria lukuja, siksi kysymysmerkki perässä.

Jokatapauksessa äkkijyrkkyys tulee horisontin lähellä, ja se on se mysteeri. Se on ihan oikea asia mutta väärässä kohtaa. Oikea asia sikäli että mustan aukon massa on singulariteetissa, ainakin "säteilijän näkökulmasta", ja vasta singulariteetin lähellä pitäisi olla äkkijyrkkää. Horisontti on tässä laskurissa vielä 3 km päässä singulariteetista, joten tällä korkeudella ei vielä pitäisi olla erityisen jyrkkää. Tässä siis tarkastelu g-kentän ollessa muuttuja.

Mutta jos jätetään g-kenttä rauhaan ja muuttujaksi valitaan aikadilataatio, niin silloin on helpompi ymmärtää punasiirtymän meno äärettömyyteen horisontissa.
Mutta tässäkin tulee ongelma: alemmassa laskurissa havaitsija on metrin korkeudella h:sta mutta ei voi nähdä h:n sisäpuolelle vaikka potentiaaliero on suht pieni (g-kenttä siis ei tässä tarkastelussa ole ääretön h:ssa).

Siispä molemmissa muuttujissa, sekä g-kentässä että dilataatiossa, tulee ongelma.

Olisiko sitten ratkaisu pituuskontraktiossa? Eli tapahtuuko horisontin lähellä kontraktiota joka selittäisi ongelmat?

jussi_k_kojootti

Lainaus käyttäjältä: mistral - 26.03.2017, 01:21:41
Ymmärrän tämän laskurin niin että punasiirtymä lähestyy ääretöntä äkkijyrkästi horisontissa.
Alimmalla rivillä säteilijä on jossain millin miljardisosien(?) päässä horisontista ja Maassa mitataan aallonpituudeksi 54mrd(?) kilometriä. En muista noita suuria lukuja, siksi kysymysmerkki perässä.

Punasiirtymä, fraktionaalinen aallonpituuden tai taajuuden muutos.  Ei ne mitään kilometrejä ole.  Siinä f1-sarakkeessa näkyy punasiirtynyt taajuus, josta voit laskea vastaavan aallonpituuden kaavasta

c = f * L  <=>  L = c / f

c valonnopeus, f taajuus, L aallonpituus

Lainaa
Mutta jos jätetään g-kenttä rauhaan ja muuttujaksi valitaan aikadilataatio, niin silloin on helpompi ymmärtää punasiirtymän meno äärettömyyteen horisontissa.

Punasiirtymän ja aikadilataatio-kertoimen eli lorentzin gamman välillä vallitsee tässä kohtaa yksinkertainen relaatio,

z + 1 = gamma

jussi kantola / oulun arktos
CG-5 GOTO + KWIQ-guiding + SW80ED  // 10" dobson // canon eos 450d mod & 400d / ASI 120MM
http://astrobin.com/users/jussi_k_kojootti/
http://oulunarktos.fi/

Eusa

Tässä on niin mainiosti päädytty oikean ongelman äärelle, joten en malta olla kuvailematta yritettäni horisonttiongelman ratkaisuksi.

Postuloidaan avaruusajan kaarevuuksien välinen vuorovaikutus - ei siis pelkästään massakappaleiden välinen. Vuorovaikutus huolehtii gravitaatiomuutosten välittymisestä kaarevuusmuutoksiksi avaruusajassa vaikutusnopeudella c.

Valmiiksi pallogeometrisissa järjestelmissä, kuten aurinkokunta, vähäiset muutokset pallogeometriseen kaarevuuteen välittyvät käytännössä kertavuorovaikutuksella. Vain ehkä aurinkokunnan laitamilla ja nopeissa ohilennoissa voi esiintyä mitattavia sarjallisia vaikutuksia.

Galakseissa pitkillä etäisyyksillä sarjalliset vuorovaikutusviiveet kertautuvat merkittäväksi energiasisällöksi, pimeäksi massaksi.

Mustaksi aukoksi romahtamisen käynnistyessä gravitaation kaarevuusmuutoksia pyrkii tapahtumaan runsaasti. Galaksin keskustan massa romahtaa tiiviimmälle alueelle ja keskiön ulkopuolelle jää viiveiden johdosta kaarevuutta, joka vuorovaikuttaa sauvan ja kierteishaarojen kanssa, niin, että keskiön pyöriminen hidastuu ja kierteishaarojen nopeutuu. Dynamiikasta tulee elliptisen kaarevuuden pysyvä kenttärakenne, kaarevuus ei pysty koskaan tasoittumaan pallogeometriseksi galaksin pyörimisen aiheuttaman "pakenemisen" vuoksi.

Tullaksemme horisontin ja mustan aukon muodostumiseen:

Vaikka säteilypaine ei pysty vastustamaan romahdusta, pimeän massan mekanismi huolehtisi luonnostaan siitä, ettei kaarevuusjyrkkyys missään yllä 45 asteeseen, vaan romahduksessa kappaleet miniatyyristyvät Shapiro-efektin mukaisesti, syntyy lisää paikallista tilaa, mikä osaltaan "pullauttaa" kokonaiskaarevuutta laajemmalle alueelle.

Tämmöisen hypoteesin mukaan ei siis aitoja mustia aukkoja ja niihin liittyviä horisontteja eikä singulariteettejakaan olisi olemassa.

jussi_k_kojootti

Lainaus käyttäjältä: Eusa - 26.03.2017, 08:54:44
Tämmöisen hypoteesin mukaan ei siis aitoja mustia aukkoja ja niihin liittyviä horisontteja eikä singulariteettejakaan olisi olemassa.

Jaa.  Kumma ettei Sgr A* näy infrapunassa, vaikka loistaa hypoteesissäsi kuin 4 miljoonaa aurinkoa.  Siitä viereltä kyllä kyetään havaitsemaan yksittäisiä tähtiä.   :rolleyes:
jussi kantola / oulun arktos
CG-5 GOTO + KWIQ-guiding + SW80ED  // 10" dobson // canon eos 450d mod & 400d / ASI 120MM
http://astrobin.com/users/jussi_k_kojootti/
http://oulunarktos.fi/

Pappis

Lainaus käyttäjältä: Eusa - 26.03.2017, 08:54:44
Tämmöisen hypoteesin mukaan ei siis aitoja mustia aukkoja ja niihin liittyviä horisontteja eikä singulariteettejakaan olisi olemassa.
Eikä galaksejakaan, kun ne hajoaisivat yhä kiihtyvän pyörimisen takia?!

Eusa

Lainaus käyttäjältä: ketarax - 26.03.2017, 09:07:34
Jaa.  Kumma ettei Sgr A* näy infrapunassa, vaikka loistaa hypoteesissäsi kuin 4 miljoonaa aurinkoa.  Siitä viereltä kyllä kyetään havaitsemaan yksittäisiä tähtiä.   :rolleyes:
Sm-säteilyn energia kuluisi laajentuneen avaruustilan ylläpitoon eli olisi mustan kappaleen tasapainossa.

Eusa

Lainaus käyttäjältä: Pappis - 26.03.2017, 10:28:52
Eikä galaksejakaan, kun ne hajoaisivat yhä kiihtyvän pyörimisen takia?!
Ei vaan galaksin keskustasta laajemmalle "pullahtanut" kaarevuus, pimeä massa, pitää galaksin kasassa.

mistral

Lainaus käyttäjältä: ketarax - 26.03.2017, 08:26:04
Punasiirtymä, fraktionaalinen aallonpituuden tai taajuuden muutos.  Ei ne mitään kilometrejä ole.  Siinä f1-sarakkeessa näkyy punasiirtynyt taajuus, josta voit laskea vastaavan aallonpituuden kaavasta

c = f * L  <=>  L = c / f

c valonnopeus, f taajuus, L aallonpituus

Punasiirtymän ja aikadilataatio-kertoimen eli lorentzin gamman välillä vallitsee tässä kohtaa yksinkertainen relaatio,

z + 1 = gamma

Oho, luin sen viime yönä väärin. Onko oikea aallonpituus alimmassa rivissä 163mm?

Kuitenkin tulkitsen laskurin niin että säteilijän lähestyessä horisonttia, Maassa mitattu aallonpituus menee äärettömäksi. Onko oikea tulkinta?

mistral

Lainaus käyttäjältä: Joksa - 26.03.2017, 13:41:38
Valo kulkee 0-ajan koordinaatistossa joten pysötynyt aika ei pysäytä valoa,

Joo, ei tietenkään valo paikan päällä pysähdy vaan Maan suhteen.

jussi_k_kojootti

Lainaus käyttäjältä: mistral - 26.03.2017, 13:40:01
Kuitenkin tulkitsen laskurin niin että säteilijän lähestyessä horisonttia, Maassa mitattu aallonpituus menee äärettömäksi. Onko oikea tulkinta?

Ei, koska laskuri ei lähetä fotonia horisontilta asti.  Vasta horisontista, schwarzschildin säteen etäisyydeltä, eikä tippaakaan yli, punasiirtymä on ääretön.

Tässä suunnilleen niin läheltä kuin onnistuu.  Voit koettaa vielä lähempääkin, mutta tulokset voi olla vääriä / pakotettuja.

http://rubor.org/schwarzschildarb.php?d=1e-53&p1e=1e-53

(Tämä on niin lähellä, että jos yrittäisit sovittaa planckin mittaa ao. etäisyydelle, olisi se sama kuin että onnistuisit sijoittamaan rantapallon tarkasti protonin halkaisijan kymmenesmiljoonasosan päähän jostain.)

Mutta siis ON oikea tulkinta sikäli että juu juu, divergoi divergoi, äärettömiin pasahtaa.  Siksi se on niin mielenkiintoinen kohta:  tapahtumahorisontti, näennäinen horisontti, aeva sama, mutta jännä on.
jussi kantola / oulun arktos
CG-5 GOTO + KWIQ-guiding + SW80ED  // 10" dobson // canon eos 450d mod & 400d / ASI 120MM
http://astrobin.com/users/jussi_k_kojootti/
http://oulunarktos.fi/